Reproductive Disorders Practice Questions

Lakukan tugas rumah & ujian kamu dengan baik sekarang menggunakan Quizwiz!

The client is diagnosed with breast cancer and is considering whether to have a lumpectomy or a more invasive procedure, a modified radical mastectomy. Which information should the nurse discuss with the client? Select all that apply. 1. Ask if the client is afraid of having general anesthesia. 2. Determine how the client feels about radiation and chemotherapy. 3. Tell the client she will need reconstruction with either procedure. 4. Find out if the client has any history of breast cancer in her family. 5. Inform the client of no difference in overall survival rate between procedures.

Answer: 2, 5 1. General anesthesia is used for either procedure. 2. The client should understand the treatment regimen for follow-up care. A lumpectomy requires follow-up with radiation therapy to the breast and then systemic chemotherapy. In a modified radical mastectomy, radiation therapy is not always required, but systemic chemotherapy is the usual treatment. 3. A lumpectomy removes only the tumor and a small amount of tissue surrounding the tumor; reconstruction is not needed. 4. A history of breast cancer in the family is immaterial because this client has breast cancer. 5. If the cancer is in its early stages, this regimen has results equal to those with a modified radical mastectomy. TEST-TAKING HINT: The test taker should use the nursing process to answer this question and select an assessment intervention, which eliminates option "3" as a correct answer. Option "1" uses the word "afraid," which is an assumption; therefore, this option could be eliminated.

The nurse is preparing the client for the insertion of a pessary. Which information should the nurse teach the client? Select all that apply. 1. The pessary does not need to be changed. 2. The client should clean the pessary routinely. 3. The pessary must be inserted in surgery. 4. Estrogen cream is necessary for effective use of a pessary. 5. Routine follow-up visits are recommended every 3 months.

Answer: 2, 5 1. Pessaries need to be changed to prevent complications. 2. Clients do need to clean the pessary routinely. 3. Pessaries are not only inserted in surgery, but also in such places as an HCP's office. 4. Hormone cream may be used and prescribed by the HCP. 5. Routine follow-up visits are required to evaluate the health of the vaginal tissue. TEST-TAKING HINT: The test taker should realize any removable device inserted into the body requires routine changing at some time; therefore, option "1" can be eliminated. Option "4" is a medication and a pessary is a device, so option "4" can be eliminated.

The client scheduled for a radical prostatectomy surgical procedure has an intravenous antibiotic medication ordered on call to surgery. The antibiotic is prepared in 100 mL of sterile normal saline. At what rate should the nurse infuse, via the IV pump, the medication in 30 minutes?

Answer: 200 mL/hr IV pumps work on the principle of the number of milliliters per hour to infuse, and unless otherwise specified, IVPB medications are infused over a 30-minute time frame. Sixty minutes in 1 hour divided by 30 = 2. Two times the volume of 100 mL = 200 mL, the rate at which the nurse should set the pump. TEST-TAKING HINT: The test taker must know basic rules for medication administration and be able to compute simple math equations.

The client had a right modified radical mastectomy 4 years before and is now being admitted for a cardiac work-up for chest pain. Which intervention is most important for the nurse to implement? 1. Determine when the client had chemotherapy last. 2. Ask the client if she received doxorubicin. 3. Post a sign at the head of bed for staff not to use the right arm for venipunctures or BPs. 4. Examine the chest wall for cancer sites.

Answer: 3 1. A client 4 years post-mastectomy should be finished with adjuvant therapy, which lasts from 6 months to 1 year. 2. The client may have received doxorubicin (Adriamycin), an antineoplastic agent that is a cardiotoxic medication, but knowing this will not change the tests performed or preparation for the tests. 3. The nurse should post a message at the head of the client's bed to not use the right arm for blood pressures or laboratory draws. This client is at risk for lymphedema, and this is a lymphedema precaution. 4. The chest wall is sometimes involved in breast cancer, but the most important intervention is to prevent harm to the client. TEST-TAKING HINT: The question is asking for an intervention common in the health-care industry. There are many breast cancer survivors with unrelated problems and comorbidities, but the nurse must still be aware of the lingering needs of the client.

The outpatient clinic nurse is working with clients diagnosed with STIs. Which long-term complication should the nurse discuss with the clients about STIs? 1. Stress the need for clients to finish all antibiotic prescriptions completely. 2. Inform the clients that, legally, many STIs must be reported to the health department. 3. Sexually transmitted infections can result in reproductive problems. 4. Discuss the myth that acquired immunodeficiency syndrome is an STI.

Answer: 3 1. A general rule when discussing antibiotics is to teach clients to finish all of the prescription, but this is not specific information related to STIs. 2. Most STIs must be reported to comply with public health laws, but this is not a long-term complication of having an STI. 3. Because of the scarring of reproductive tissue, infertility may be an issue resulting from STI infection. 4. AIDS is a sexually transmitted infection and can also be transmitted by non-sexual blood and body fluid exposure. TEST-TAKING HINT: The test taker can rule out option "1" because of the generalized nature of the option; it is not specific to STIs. Option "2" does not address a long-term complication for the client. And option "4" requires the test taker to know the transmission of the disease.

The nurse is caring for a 30-year-old nulliparous client reporting severe dysmenorrhea. Which diagnostic test should the nurse prepare the client to undergo to determine the diagnosis? 1. A bimanual vaginal exam. 2. A pregnancy test. 3. An exploratory laparoscopy. 4. An ovarian biopsy.

Answer: 3 1. A vaginal examination does not provide a definitive diagnosis to determine the cause of the pain. 2. A pregnancy test is not usually ordered unless the client has a reason to think she may be pregnant. Pregnancy temporarily alleviates the symptoms of endometriosis because neither ovulation nor menses occur during pregnancy. 3. There is a high incidence of endometriosis among women with no biological children (nulliparity) and those having children later in life. The most common way to diagnose this condition is through an exploratory laparoscopy. 4. The ovaries lie deep within the pelvic cavity. Some form of abdominal procedure must be performed, such as laparoscopy, to reach the ovaries. However, the symptoms are not those of an ovarian cyst. TEST-TAKING HINT: The test taker could eliminate answer option "1" because "diagnosis" is in the stem. The stem is asking for a procedure providing a definitive diagnosis. Option "2" could be eliminated because, if the client is menstruating (dysmenorrhea means "painful menstruation"), then the client is usually not pregnant.

The nurse is teaching men about early detection of prostate cancer according to the American Cancer Society (ACS) guidelines. Which should the nurse teach the clients? 1. Beginning at age 39, men should have a digital rectal examination (DRE) followed by a prostate-specific antigen (PSA). 2. Beginning at age 45, men should have a rectal sonogram, and if positive, a DRE. 3. Beginning at age 50, men should have a prostate-specific antigen (PSA) followed by a DRE. 4. Beginning at age 60, men should have a PSA followed, if positive, by a prostate biopsy.

Answer: 3 1. According to the ACS, beginning at age 50, a PSA level followed by a DRE should be done yearly. 2. According to the ACS, beginning at age 50, a PSA level followed by a DRE should be done yearly. 3. According to the ACS, beginning at age 50, a PSA level followed by a DRE should be done yearly. 4. According to the ACS, beginning at age 50, a PSA level followed by a DRE should be done yearly. TEST-TAKING HINT: The test taker must have a level of knowledge concerning basic nursing activities and information to teach a client.

The nurse writes a problem of "anticipatory grieving" for a client diagnosed with ovarian cancer. Which nursing intervention is a priority for this client? 1. Request the HCP to order an antidepressant medication. 2. Refer the client to a CanSurmount volunteer for counseling. 3. Encourage the client to verbalize feelings about having cancer. 4. Give the client an advance directive form to fill out.

Answer: 3 1. An antidepressant may be needed at some time, but at this point the nurse should offer time, interest, and encouragement for the client to discuss the feeling of having cancer. 2. CanSurmount volunteers are extremely helpful in talking about having cancer with the client, but they do not provide counseling. The programs work based on the fact a cancer survivor after going through treatment can relate to the client beginning treatment. 3. The nurse should plan to spend time with the client and allow the client to discuss the feelings of having cancer, dying, fear of the treatments, and any other concerns. 4. The client will need to complete an advance directive, but this action does not address the client's grieving process. TEST-TAKING HINT: The test taker could eliminate option "2" because a client is not referred to a volunteer for counseling. Only one option directly addresses the problem and requires the nurse to interact with the client.

The young female client is admitted with pelvic inflammatory disease secondary to a chlamydia infection. Which discharge instruction should be taught to the client? 1. The client will develop antibodies to protect against future infection. 2. This infection will not have any long-term effects for the client. 3. Both the client and the sexual partner must be treated simultaneously. 4. Once the infection subsides, the pain will go away and not be a problem.

Answer: 3 1. Chlamydia does not cause an antigen-antibody reaction. 2. There are long-term problems associated with any STI. Chlamydia may have the long-term effects of chronic pain and increased risk for ectopic pregnancy, postpartum endometritis, and infertility. 3. If both the client and the sexual partner are not treated simultaneously, the sexual partner can reinfect the client. 4. The client may develop chronic pelvic pain as a result of the infection. TEST-TAKING HINT: Options "2" and "4" have a form of absolute. The words "any," "will," or "will not" are absolutes, and in health care, there are very few absolutes.

The day nurse is administering the initial dose of medications to a newly admitted client at 0900. Which medication should the nurse administer first? Diagnosis: Prostatitis - Digoxin 0.125 mg PO every day - Furosemide 40 mg every 12 hours - Gentamycin 80 mg IVPB every 6 hours - Acetaminophen 650 mg every 4 hours prn 1. Digoxin. 2. Furosemide. 3. Gentamycin. 4. Acetaminophen.

Answer: 3 1. Digoxin is a routine maintenance medication, and the nurse can begin the medication from 0800 to 1000, an hour before or after the 0900 time scheduled. 2. Furosemide is a routine maintenance medication, and the nurse can begin the medication from 0800 to 1000, an hour before or after the 0900 time scheduled. 3. An initial dose of IV antibiotics should be considered a STAT or Now medication. The nurse should make sure that any ordered cultures are obtained and the medication initiated within 1 hour of the prescription being written. 4. Acetaminophen is a mild analgesic or antipyretic medication; this could be administered if needed after the antibiotic. TEST-TAKING HINT: The test taker could eliminate options "1" and "2" because the guidelines allow up to 1 hour after the scheduled time to administer these medications.

The occupational health nurse is preparing a class regarding sexually transmitted infections (STIs) for employees at a manufacturing plant. Which high-risk behavior information should be included in the class information? 1. Engaging in oral or anal sex decreases the risk of getting an STI. 2. Using a sterile needle guarantees the client will not get an STI. 3. The more sexual partners, the greater the chance of developing an STI. 4. If a condom is used, the client will not get a sexually transmitted infection.

Answer: 3 1. Engaging in oral and anal sex increases the risk of contracting an STI. 2. Using a sterile needle for drug abuse ensures the client will not get an STI from needle sharing, but the client can still contract an STI from other risky behaviors. 3. The more sexual partners, the greater the risk of contracting an STI. 4. Condom use provides a barrier to contracting an STI, but it is not a guarantee. The condom can break or come off during intercourse. TEST-TAKING HINT: In option "2" the word "guarantees" appears, and the nurse cannot guarantee anything in dealing with healthcare issues. Option "4" is an absolute statement—"will not get"—and can be eliminated on this basis.

The RN and an UAP are caring for clients on a genitourinary floor. Which nursing task can be delegated to the UAP? 1. Increase the drip rate on a continuous bladder irrigation set. 2. Check the suprapubic catheter insertion site for infection. 3. Encourage the 2-hour postoperative client to turn and cough. 4. Document the amount of red drainage in the catheter.

Answer: 3 1. Increasing the drip rate on a continuous bladder (Murphy) irrigation requires nursing judgment and cannot be delegated. 2. In this situation, checking for infection is an assessment situation. The nurse cannot delegate assessment. 3. The UAP can be asked to help a client turn, cough, and deep breathe. This requires the UAP to perform an action only, not to use judgment or to assess. 4. Red drainage in the catheter implies bleeding. The nurse should assess the amount of bleeding occurring. TEST-TAKING HINT: The test taker should choose the lowest level activity when choosing an action to have UAP perform. Blood should always be assessed.

The client is diagnosed with tertiary syphilis. Which clinical manifestations should the nurse expect the client to exhibit? 1. Lymphadenopathy and hair loss. 2. Warts in the genital area. 3. Dementia and psychosis. 4. Raised rash covering the body.

Answer: 3 1. Lymphadenopathy and hair loss are clinical manifestations of secondary syphilis, not tertiary syphilis. 2. Genital warts are not signs of tertiary syphilis. 3. Aortitis and neurosyphilis (dementia, psychosis, stroke, paresis, and meningitis) are the most common manifestations of tertiary syphilis. 4. A rash covering the body is a symptom of gonorrhea. TEST-TAKING HINT: The keyword in this question is "tertiary." The test taker must decide which disease has three distinct phases and then which clinical manifestations accompany each phase.

The nurse is assessing a client admitted for possible testicular cancer. Which assessment data support the client having testicular cancer? 1. The client reports pain when urinating. 2. There is a chancre sore on the shaft of the penis. 3. The client reports heaviness in the scrotum. 4. There is a red, raised rash on the testes.

Answer: 3 1. Pain when urinating indicates a urinary tract infection. 2. A chancre sore on the shaft of the penis indicates syphilis, a sexually transmitted infection. 3. Classic clinical manifestations of cancer of the testes are a mass on the testicle, painless enlargement of the testes, and heaviness of the scrotum or lower abdomen. 4. There is no rash associated with cancer of the testes. TEST-TAKING HINT: The test taker could eliminate option "2" because the penis and testes are separate body parts. The testicles lie within the scrotum. Only one option concerns that part of the body.

The nurse is working in a health clinic. Which condition is required to be reported to the public health department? 1. Pelvic inflammatory disease. 2. Epididymitis. 3. Syphilis. 4. Ectopic pregnancy.

Answer: 3 1. Pelvic inflammatory disease (PID) does not have to be reported, but the cause of the PID may need to be reported. 2. There are causes for epididymitis other than an STI. 3. Syphilis is an STI and therefore must be reported to the appropriate health department. 4. An ectopic pregnancy may have numerous causes. TEST-TAKING HINT: Only one answer option is an STI. The other diseases and conditions may be caused by STIs, but they all have other causes as well.

The 80-year-old male client has been diagnosed with cancer of the prostate. Which treatment should the nurse discuss with the client? 1. Radiation therapy every day for 4 weeks. 2. Radical prostatectomy with lymph node dissection. 3. Gonadotropin-releasing hormone agonists (GnRH). 4. Penile implants to maintain sexual functioning.

Answer: 3 1. Radiation therapy is considered aggressive therapy and is not recommended for the older client unless needed to alleviate pain from bony metastasis. 2. A radical prostatectomy with lymph node dissection is extensive surgery and only recommended for clients with a life expectancy of greater than 10 years. 3. Gonadotropin-releasing hormone agonists (GnRH) such as leuprolide (Lupron) lower the amount of testosterone made by the testicles and slows the growth of the tumor. Some men with a life expectancy of fewer than 10 years choose not to treat cancer at all and will usually die from causes other than prostate cancer. 4. Penile implants do not treat prostate cancer. Sexual functioning may or may not be impaired, depending on whether the client is treated surgically by radical prostatectomy. If so, then the client may be prescribed Viagra or Cialis. Eighty-year-old men are not candidates for a radical prostatectomy. TEST-TAKING HINT: The test taker should notice the age of the client. When an age is provided in the question, it is significant. Older clients do not tolerate many treatments well, so the test taker should choose the least invasive treatment.

The client has had an exploratory laparotomy to remove an ovarian tumor. The pathology report classifies the tumor as a "low malignancy potential" tumor. Which statement explains the scientific rationale for this pathology report? 1. The client does not have cancer but will need adjuvant therapy. 2. The client would have developed cancer if the tumor had not been removed. 3. These borderline tumors resemble ovarian cancer but have better outcomes. 4. The client has a very poor prognosis and has less than 6 months to live.

Answer: 3 1. The client has low-grade cancer occurring in a small percentage of ovarian tumors. The affected ovary usually is removed, and the client may or may not require additional treatment. Women with this type of tumor are usually younger than age 40 years. 2. The tumor is classified as cancer. The follow-up care is not as extensive because of the characteristics the tumor displays. 3. Ovarian low malignant potential tumors have abnormal cells in the tissues covering the ovary. These tumors are low-grade cancers with fewer propensities for metastasis than most ovarian cancers. 4. This client has a better prognosis than 85% of clients diagnosed with ovarian cancer. TEST-TAKING HINT: The test taker could eliminate option "4" because "low malignancy potential" and "poor prognosis" do not match. The statement in option "1" says the client does not have cancer but will need therapy for cancer, so the test taker could eliminate this option.

The client diagnosed with endometriosis experiences pain rated a "5" on a 1-to-10 pain scale during her menses. Which intervention should the nurse teach the client? 1. Teach the client to take a stool softener when taking morphine. 2. Instruct the client to soak in a tepid bath for 30 to 45 minutes when the pain occurs. 3. Explain the need to take NSAIDs with food. 4. Discuss the possibility of a hysterectomy to help relieve the pain.

Answer: 3 1. The client taking a narcotic medication should be placed on a bowel regimen, but this client would not be prescribed morphine, a narcotic medication. 2. A tepid bath for 30 to 45 minutes is not appropriate because the lukewarm water gets cold. A heating pad to the abdomen sometimes helps with the pain. 3. The medication of choice for mild to moderate dysmenorrhea is an NSAID. NSAIDs cause gastrointestinal upset and should be taken with food. 4. This may be an option eventually, but the stem did not give an age nor state the client has decided she does not want to get pregnant. TEST-TAKING HINT: The test taker should not read into the question. Option "4" is only correct when more information is provided. The test taker must know about the scales used to rate pain, nausea, or depression. The client's report of midrange symptoms does not indicate the need for routine narcotic administration.

The client diagnosed with ovarian cancer is prescribed radiation therapy for regional control of the disease. Which statement indicates the client requires further teaching? 1. "I will not wash the marks off my abdomen." 2. "I will have a treatment every day for 6 weeks." 3. "Nausea caused by radiation therapy cannot be controlled." 4. "I need to drink a nutritional shake if I don't feel like eating."

Answer: 3 1. The radiation markings on a client are there to guide the technician to irradiate only the area within the marks. The marks must remain until the client has completed the treatments. 2. Radiation therapy is administered in fractionated (divided) doses to allow for the regeneration of normal cells. Cancer cells do not regenerate as rapidly as normal cells. 3. There are many medications prescribed for cancer or treatment-induced nausea. The client should notify the HCP if adequate relief is not obtained. 4. Cancer treatments frequently interfere with the client's appetite, but supporting the nutritional status of the client is important. TEST-TAKING HINT: The question is an "except" question. All options except one will be statements indicating the client does understand the teaching. If the test taker missed the information making this an "except" question, finding two options with correct answers might clue the test taker to reread the stem.

The client has been diagnosed with cancer of the breast. Which referral is most important for the nurse to make? 1. The hospital social worker. 2. CanSurmount. 3. Reach to Recovery. 4. I Can Cope.

Answer: 3 1. The social worker assists clients in finding a nursing home placement and financial arrangements and does some work with clients to discuss feelings, but this is not the best referral. 2. CanSurmount volunteers work with all types of clients diagnosed with cancer, not just clients diagnosed with breast cancer. 3. Reach to Recovery is a specific referral program for clients diagnosed with breast cancer. 4. I Can Cope is a cancer education program for all clients diagnosed with cancer and their significant others. TEST-TAKING HINT: The question asks for the most appropriate referral, and the test taker should choose the one specific to breast cancer.

The nurse enters the room of a 24-year-old client diagnosed with testicular cancer. The fiancée of the client asks the nurse, "Will we be able to have children?" Which is the nurse's best response? 1. "Your fiancée will be able to father children like always." 2. "You will have to adopt children because he will be sterile." 3. "You and he should consider sperm banking before treatment." 4. "Have you discussed this with your fiancée? I can't discuss this with you."

Answer: 3 1. The usual treatment for testicular cancer is the removal of the involved testicle followed by radiation to the area and chemotherapy. Every attempt is made to shield the remaining testicle from the radiation, but sterility sometimes occurs. 2. With artificial insemination, the client may be able to father children, if the client has banked his sperm. 3. Sperm banking will allow the client to father children through artificial insemination with the client's sperm. 4. The nurse is in the client's room. The client's presence implies consent for the nurse to discuss his case with the fiancée. TEST-TAKING HINT: The nurse must abide by HIPAA rules. It is important for the nurse to know and understand the confidentiality laws. Options "1" and "2" are opposites, so only one could be correct or neither may be correct. Option "3" actually gives an option that lies between the extremes.

The client has had a total abdominal hysterectomy for cancer of the ovary. Which diet should the nurse discuss when providing discharge instructions? 1. A low-residue diet without seeds. 2. A low-sodium, low-fat diet with skim milk. 3. A regular diet with fruits and vegetables. 4. A full liquid-only diet with milkshake supplements.

Answer: 3 1. This diet is appropriate for a client diagnosed with diverticulitis. 2. This diet applies to a client diagnosed with coronary artery disease and hypertension. 3. The client is not placed on a specific diet, but it is always a good recommendation to include fruits and vegetables in the diet. 4. There is no reason to limit the consistency of the foods consumed to full liquids. TEST-TAKING HINT: The test taker should recognize option "3" as a recommended diet for all clients without a specific disease process limiting the types of foods consumed.

The client is diagnosed with metastatic prostate cancer to the bones. Which nursing intervention should the nurse implement? 1. Prepare for a transurethral resection of the prostate. 2. Keep the foot of the bed elevated at all times. 3. Place the client on a scheduled bowel regimen. 4. Discuss the client's altered sexual functioning.

Answer: 3 1. This intervention addresses the prostate cancer but not the metastatic process of bone involvement. 2. There is no reason to keep the foot of the bed elevated. 3. Bone metastasis is very painful, and the client should be placed on a scheduled regimen of pain medication. Pain medication slows peristalsis and causes constipation. The client should be placed on a routine bowel management program to prevent impactions. 4. This does not address the metastasis to the bone. TEST-TAKING HINT: The test taker must decide which intervention addresses both the cancer and the metastasis to the bone. Only one option does this.

The nurse is preparing the care plan for a 45-year-old client after a radical prostatectomy. Which psychosocial and physiological problem should be included in the plan? 1. Altered coping. 2. High risk for hemorrhage. 3. Sexual impotence. 4. Risk for electrolyte imbalance.

Answer: 3 1. This is a psychosocial problem. 2. This is a physiological problem. 3. This problem has both physiological and psychosocial implications. 4. This is a physiological problem. TEST-TAKING HINT: The test taker should read the stem carefully. The stem asks for a physiological and psychosocial problem. Options "1," "2," and "4" can be sorted into only one of the categories.

The client frequently finds lumps in her breasts, especially around her menstrual period. Which information should the nurse teach the client regarding breast self-care? 1. This is a benign process, which does not require follow-up. 2. The client should eliminate chocolate and caffeine from the diet. 3. The client should practice breast self-examination monthly. 4. This is the way breast cancer begins, and the client needs surgery.

Answer: 3 1. This is symptomatic of benign fibrocystic disease, but follow-up is always needed if the lumps do not go away when the hormone levels change. 2. Some practitioners suggest eliminating caffeine and chocolate from the diet if the breasts become tender from the changes, but there is no research supporting this to be effective in controlling the discomfort associated with fibrocystic breasts. 3. The American Cancer Society (2015) no longer recommends breast self-examination (BSE) for all women, but it is advisable for women with known breast conditions to perform BSE monthly to detect potential cancer. 4. The client may need a breast biopsy for potential breast cancer at some point, but breast cancer develops when there is an alteration in the DNA of a cell. TEST-TAKING HINT: The test taker could eliminate option "1" because of the clause "does not require follow-up." The question is asking about self-care, and only two options—"2" and "3"—involve the client doing something. The test taker should choose between these.

The RN charge nurse is making rounds on the genitourinary surgery floor. Which action by the primary nurse warrants immediate intervention? 1. The nurse elevates the scrotum of a client after an orchiectomy. 2. The nurse encourages the client to cough, although he reports pain. 3. The nurse empties the client's JP drain and leaves it rounded. 4. The nurse asks the UAP to empty a catheter drainage bag.

Answer: 3 1. This should be done for clients after scrotal surgery. 2. Postoperative clients do not want to perform deep-breathing and coughing exercises because it hurts, but they should be encouraged to do so to prevent complications. 3. The Jackson Pratt (JP) drain is a drain attached to a bulb, and the bulb should remain compressed to apply gentle suction to the surgical site. 4. This is an appropriate delegation. TEST-TAKING HINT: The test taker could eliminate option "4" by knowing the rules of delegation. The RN did not ask the UAP to perform anything requiring nursing judgment or assessment. Option "2" is just good nursing practice and could be eliminated. The charge nurse does not need to stop this action.

Which recommendation is the American Cancer Society's (ACS) 2015 guideline for the early detection of breast cancer? 1. Beginning at age 18, have a biannual clinical breast examination by an HCP. 2. Beginning at age 30, perform monthly breast self-examinations. 3. At age 45 through 54, receive a yearly mammogram. 4. Beginning at age 50, have a breast sonogram every 5 years.

Answer: 3 1. Unless there is a personal history of breast cancer or strong family history, clinical breast examinations are not recommended for average risk women at any age (American Cancer Society, 2015). 2. If the client is going to perform breast self-examination (BSE), it should begin at age 18. The ACS no longer includes monthly BSE as part of its guidelines. 3. The ACS recommends a yearly mammogram for the early detection of breast cancer beginning at age 45 to age 54 and approximately every 2 years at the age of 55 and older. Before age 45 and after age 54 it should be a discussion between the woman and her HCP to determine if more frequent mammograms are warranted. A mammogram can detect disease that will not be large enough to feel. 4. Breast sonograms are performed to diagnose specific breast disease when a screening mammogram has shown a suspicious area. TEST-TAKING HINT: This is a knowledge-based question. The test taker might be swayed by the option about BSE, but the age must be considered.

The nurse is formulating a care plan for a client who had an abdominal hysterectomy. Which nursing diagnosis is appropriate for the client developing a complication? 1. Potential for urinary retention. 2. Potential for nerve damage. 3. Potential for intestinal obstruction. 4. Potential for fluid imbalance.

Answer: 3 1. Urinary retention is a complication of a vaginal hysterectomy. 2. Nerve damage is a possible complication of improper positioning during surgery, but the client is "post" surgery. 3. Clients after a total abdominal hysterectomy are at risk for intestinal obstruction. 4. An imbalance of fluid is a complication of several surgeries, not specifically a total abdominal hysterectomy. TEST-TAKING HINT: When the test taker does not know the answer, the test taker should consider the similarities and the differences in surgeries. A total abdominal hysterectomy has complications similar to all types of abdominal surgeries.

What intervention should the nurse implement for a client diagnosed with a rectocele? Select all that apply. 1. Limit oral intake to decrease voiding. 2. Encourage a low-residue diet. 3. Administer a stool softener daily. 4. Arrange for the client to take sitz baths. 5. Recommend pelvic floor exercises.

Answer: 3, 5 1. There is no reason to limit oral intake or to decrease voiding. 2. The client should be eating a high-fiber diet to prevent constipation. 3. Stool softeners and laxatives are used to prevent and treat constipation, which is common with a rectocele. Because of the positioning of the rectum, the stool can stay in the rectal pouch, causing constipation 4. Sitz baths are not used to treat a rectocele. 5. Pelvic floor exercises, also known as Kegel exercises, can strengthen the pelvic floor muscles to decrease symptoms. TEST-TAKING HINT: The test taker knowledgeable of medical terminology could eliminate option "1," which has the word "voiding" because "rectocele" refers to the rectum.

The nurse is assessing a male client for clinical manifestations of gonorrhea. Which data support the diagnosis? 1. Presence of a chancre sore on the penis. 2. No symptoms. 3. A CD4 count of less than 200. 4. Pain in the testes and scrotal edema.

Answer: 4 1. A chancre sore is a symptom of syphilis, not gonorrhea. 2. Gonorrhea is more likely to be asymptomatic in females. 3. A CD4 count of less than 200 is a diagnostic indicator for AIDS. 4. Pain in the testes and scrotal edema can indicate epididymitis, an inflammatory process of the epididymis. This and urethritis are the most common presenting symptoms in a male with gonorrhea. TEST-TAKING HINT: Two answer options mention male anatomy. If the test taker did not know the information, then choosing between these two options might be the appropriate method of elimination.

The nurse is caring for a young adult client diagnosed with gonorrhea. Which statement reflects an understanding of the transmission of STIs? 1. Only lower socioeconomic income people are at risk for gonorrhea and syphilis. 2. The longer a client waits to become sexually active, the greater the risk for an STI. 3. Females can transmit infectious diseases more rapidly than males. 4. If a client is diagnosed with an STI, the client should be evaluated for other STIs.

Answer: 4 1. All socioeconomic levels of clients contract STIs. 2. The longer the client abstains from sexual activity and the fewer partners the client has, the less the risk of an STI. 3. Females and males can spread STIs equally. Specific diseases may be asymptomatic in the sexes (in females, gonorrhea; in males, chlamydia) and they can transmit them unknowingly. 4. If the client has one STI, there is a great likelihood the client has another disease also. If one STI is found, the client should be monitored for others. TEST-TAKING HINT: Option "2" does not make sense: If sexual activity is put off, there cannot be an increased risk. Socioeconomic reasons may be a reason for delaying the treatment of a disease, but diseases are not financially based and occur in all socioeconomic levels.

The client diagnosed with ovarian cancer has had eight courses of chemotherapy. Which laboratory data warrant immediate intervention by the nurse? - RBCs: 5 - ANC: 3,500 - Platelets: 150 - WBCs: 100 1. Absolute neutrophil count. 2. Platelet count. 3. Red blood cell count. 4. Urinalysis report white blood cells.

Answer: 4 1. An absolute neutrophil count of 3,500 indicates the client has sufficient mature white blood cells, or granulocytes, to act as a defense against infections. 2. A platelet count of 150,000 is within normal range (150 × 10^3 [1,000] = 150,000). Thrombocytopenia is less than 100,000. 3. A red blood cell count of 5,000,000 is within normal limits (5 × 10^6 [1,000,000] = 5,000,000). 4. A normal urinalysis contains one to two WBCs. A report of 100 WBCs indicates the presence of an infection. A clean voided specimen should be obtained, and a urine culture should be done. This client should be prescribed antibiotics immediately. TEST-TAKING HINT: The test taker should memorize normal values for common laboratory tests. Urine will not have a large number of white blood cells unless there is a pathological process occurring. The kidneys filter the blood but do not process the destruction of blood cells.

Which tumor marker information is used to follow the progress of a client diagnosed with testicular cancer? 1. CA-125. 2. Carcinogenic embryonic antigen (CEA). 3. DNA ploidy test. 4. Human chorionic gonadotropin (hCG).

Answer: 4 1. CA-125 is a tumor marker used for a number of cancers but not for testicular cancer. 2. CEA is a nonspecific tumor marker used for a number of cancers, but it does not apply to testicular cancer. 3. The DNA ploidy test is used to determine the number of chromosomes and the arrangement of chromosomes in a tumor cell. It is used for prognosis in some cancers, such as breast cancer, but not as a tumor marker to determine response to treatment. 4. Tumor markers are substances synthesized by the tumor and released into the bloodstream. They can be used to follow the progress of the disease. Testicular cancers secrete hCG and alpha-fetoprotein. TEST-TAKING HINT: This is a knowledge-based question. If the test taker were aware of what the test tells the HCP, then the DNA ploidy test could be eliminated. The other options are all tumor markers.

The client diagnosed with gestational diabetes delivered a 10-pound 5-ounce infant. Which is a priority for the nursery nurse to monitor? 1. Failure to latch on to the breast during feeding. 2. Jaundice and clay-colored stools. 3. Parchment-like skin and lack of lanugo. 4. Low blood glucose readings.

Answer: 4 1. Failure of the baby to latch onto the breasts is not the priority for the nurse to assess. 2. There is nothing that indicates this baby will have jaundice and clay-colored stools. 3. The baby would have parchment-like skin and lack lanugo if the baby were over 40 weeks gestation. This is not stated in the stem. 4. The neonate is a high birth weight, and the mother had gestational diabetes. This infant had a high glucose content passing through the placenta in utero, and the infant's pancreas has been producing insulin to take care of the glucose content of the blood. The infant's pancreas must adjust to lower levels of glucose in the system. TEST-TAKING HINT: The test taker could eliminate option "2" because this is associated with liver or gallbladder issues, and basic knowledge of the pathophysiology of post-maturity gestation could eliminate option "3."

The nurse is teaching a class on breast health to a group of women at a senior citizen's center. Which risk factor is the most important to emphasize to this group? 1. The clients should find out about their family history of breast cancer. 2. Men at this age can get breast cancer also and should be screened. 3. Monthly breast self-examination is the key to early detection. 4. The older a woman gets, the greater the chance of developing breast cancer.

Answer: 4 1. Most women diagnosed with breast cancer have no family history of the disease. Specific genes—BRCA-1 and BRCA-2—implicated in the development of breast cancer have been identified, but most women with breast cancer do not have these genes. 2. Approximately 1,000 men are diagnosed every year with breast cancer, but as with women, it can occur at any age. Breast cancer in men frequently goes undetected because men consider this a woman's disease. 3. Mammograms can detect breast cancer earlier than BSE and are the current recommendation by the American Cancer Society. 4. The greatest risk factor for developing breast cancer is being female. The second greatest risk factor is being older. By age 80, one in every eight women develops breast cancer. TEST-TAKING HINT: The test taker cannot overlook the age when it is given in a question. "Senior citizen's center" should alert the test taker to the older age group. The test taker should decide what age has to do with the answer.

Which specific complication should the nurse assess for in the client diagnosed with a uterine prolapse recovering from an anterior and posterior repair? 1. Orthostatic hypotension. 2. Atelectasis. 3. Allen sign. 4. Deep vein thrombosis.

Answer: 4 1. Orthostatic hypotension is not a surgeryspecific complication. 2. Atelectasis is a complication of general anesthesia, not an A & P repair. 3. An Allen test is a physical examination evaluating the arterial blood supply to the radial and ulnar arteries. 4. Assessing for deep vein thrombosis (DVT) is performed on all clients having a vaginal hysterectomy. After any surgery requiring the client to be placed in the lithotomy position, the client should be assessed for DVT. These clients are at a higher risk for this complication. TEST-TAKING HINT: The test taker must have knowledge of this surgical procedure to be able to answer this question. A good choice is to choose an option assessed for most immobile clients.

The client is 8 hours post-transurethral prostatectomy for cancer of the prostate. Which nursing intervention is a priority at this time? 1. Control postoperative pain. 2. Assess abdominal dressing. 3. Encourage early ambulation to prevent DVT. 4. Monitor fluid and electrolyte balance.

Answer: 4 1. Pain does not have priority over a fluid and electrolyte imbalance. 2. There is no dressing for a transurethral resection. The body cavity is entered through the penis. 3. Early ambulation prevents several complications, but the immediate complication is electrolyte imbalance and fluid overload. 4. With irrigation of the surgical site through the indwelling three-way catheter to prevent blood clots, fluids may be absorbed through the open surgical site and retained. This can lead to fluid volume overload and electrolyte imbalance (hyponatremia). TEST-TAKING HINT: This client is 8 hours postoperative, so the test taker could eliminate preventing DVT (option "3"). Option "2" could be eliminated if the test taker looks at the surgical approach described in the procedure. Pain is a priority, but the test taker should read to see if another option has a higher priority.

Which intervention should the nurse include when teaching the client having an anterior colporrhaphy to repair a cystocele? 1. Discuss the need to perform perineal care every 4 hours. 2. Discuss the care of an indwelling catheter for at least 1 month. 3. Instruct the client how to care for the pessary inserted in surgery. 4. Teach the client how to perform Kegel exercises.

Answer: 4 1. Perineal care is given every shift or as needed. 2. The client may have an indwelling catheter for 2 to 4 days postoperative but not for a month. 3. A pessary is used in place of surgery. 4. The client should be taught how to perform Kegel exercises to strengthen the muscles. TEST-TAKING HINT: When selecting a correct answer, the test taker should always look at the adjectives, especially numbers such as four in option "1" or one in option "2."

Which statement indicates further instruction is needed for the client diagnosed with a cystocele? 1. "I need to have a sonogram to diagnose this problem." 2. "I need to practice Kegel exercises to help strengthen my muscles." 3. "I lose my urine when I sneeze because of my cystocele." 4. "I can never have sexual intercourse again."

Answer: 4 1. Sonograms and pelvic examinations are used to diagnose cystoceles. 2. Kegel exercises strengthen the pelvic floor muscles. 3. Clients diagnosed with cystoceles frequently have urinary incontinence when they cough, sneeze, laugh, lift heavy items, or make sudden jarring motions. 4. Clients diagnosed with cystoceles may have sexual intercourse unless contraindicated by another medical reason. TEST-TAKING HINT: This type of question is confusing to test takers because the correct option provides incorrect information. Most of the time absolutes such as "never," "only," and "always" make the option incorrect. Option "4" has an absolute—"never"—but it is the correct answer because all of the other options contain correct information the client has learned.

The 24-year-old female client presents to the clinic with lower abdominal pain on the left side rated as a "9" on a 1-to-10 scale. Which diagnostic procedure should the nurse prepare the client for? 1. A computed tomography scan. 2. A lumbar puncture. 3. An appendectomy. 4. A pelvic sonogram.

Answer: 4 1. The client has symptoms of an ovarian cyst, usually diagnosed by a pelvic sonogram. 2. The client has abdominal pain, not back or neurological pain, which is when a lumbar puncture is performed. 3. The appendix is in the right lower abdomen, not the left. 4. Ovarian cysts are fluid-filled sacs located on the surface of the ovary. A lower pelvic sonogram is the preferred diagnostic tool. It is not invasive and usually not painful. TEST-TAKING HINT: The test taker could eliminate options "2" and "3" by using knowledge of basic anatomy and physiology. The age of the client places this client in the typical age range for a benign ovarian cyst; before age 29 years, 98% of ovarian cysts are benign.

The client diagnosed with testicular cancer is scheduled for a unilateral orchiectomy. Which information is important to teach regarding sexual functioning? 1. The client will have ejaculation difficulties after the surgery. 2. The client will be prescribed male hormones following the surgery. 3. The client may need to have a penile implant to be able to have intercourse. 4. Libido and orgasm usually are unimpaired after this surgery.

Answer: 4 1. The client usually will be able to maintain normal sexual functioning with the remaining testicle. If not, testosterone may be prescribed to improve functioning. Ejaculation is not the problem; it is impotence (erectile dysfunction). 2. This will not be done until the HCP determines the remaining testicle is not able to maintain adequate hormone production. 3. The client will be able to function sexually with the remaining testicle alone or with prescribed male hormones. 4. Sex drive (libido) and orgasms usually are unimpaired because the client still has one testicle. TEST-TAKING HINT: The key to this question is "unilateral." The client will still have one functioning testicle.

The HCP has prescribed two IV antibiotics for the female client diagnosed with diabetes and pneumonia. Which order should the nurse request from the HCP? 1. Request written information on antibiotic-caused vaginal infections. 2. Request yogurt to be served on the client's meal trays. 3. Request a change of one of the antibiotics to an oral route. 4. Request Lactobacillus acidophilus three times a day.

Answer: 4 1. The nurse does not require an order to teach. Teaching is an independent nursing function. 2. The nurse can request the dietitian to include yogurt in the client's calorie restrictions without an order. 3. If the HCP has ordered an IV antibiotic, then there is no reason to request a change to an oral route. 4. Female clients on antibiotics are at risk for killing the good bacteria, which keep yeast infections in check. This is especially true in clients diagnosed with diabetes. L. acidophilus, a probiotic, is a yeast replacement medication. TEST-TAKING HINT: The test taker must be aware of independent nursing functions. This eliminates options "1" and "2."

The nurse writes a client problem of urinary retention for a client diagnosed with stage IV cancer of the prostate. Which intervention should the nurse implement first? 1. Catheterize the client to determine the amount of residual. 2. Encourage the client to assume a normal position for urinating. 3. Teach the client to use the Valsalva maneuver to empty the bladder. 4. Determine the client's normal voiding pattern.

Answer: 4 1. The nurse should assess the client's normal voiding pattern before taking any action. 2. This is a good intervention, but it comes after assessment. 3. The Valsalva maneuver will help the client to empty the bladder, but assessment comes before teaching. 4. Determining the client's normal voiding patterns provides a baseline for the nurse and client to use when setting goals. TEST-TAKING HINT: Assessment is the first step of the nursing process. In any question requiring the test taker to choose the first action, an answer option with the words "check," "assess," or "determine" should be considered as a possible answer.

The postmenopausal client reveals it has been several years since her last gynecological examination and states, "Oh, I don't need examinations anymore. I am beyond having children." Which statement should be the nurse's response? 1. "As long as you are not sexually active, you don't have to worry." 2. "You should be taking hormone replacement therapy now." 3. "You are beyond bearing children. How does that make you feel?" 4. "There are situations other than pregnancy that should be checked."

Answer: 4 1. This client is at risk for cancer of the ovary and uterus because of advancing age, regardless of sexual activity, and should see an HCP yearly. 2. Hormone replacement therapy (HRT) is not recommended for most postmenopausal clients because research has shown HRT increases the risk of myocardial infarctions and cerebrovascular accidents (strokes). 3. This is a therapeutic response, and the client did not state a feeling. 4. The client should have a yearly clinical examination of the breasts and pelvic area for the detection of cancer. TEST-TAKING HINT: If the stem is not asking for a therapeutic response, then factual information should be provided to the client. This eliminates option "3" as a possible answer.

The client is diagnosed with early cancer of the prostate. Which assessment data would the client report? 1. Urinary urgency and frequency. 2. Retrograde ejaculation during intercourse. 3. Low back and hip pain. 4. No problems have been noticed.

Answer: 4 1. Urgency and frequency are obstructive symptoms and are late clinical manifestations. 2. Retrograde ejaculation occurs when sperm are ejaculated into the urine; it occurs in some cases of male infertility. 3. Low back pain and hip pain are clinical manifestations of metastasis to the bone and are late symptoms. 4. In early-stage prostate cancer, the man will not be aware of the disease. Early detection is achieved by screening for cancer. TEST-TAKING HINT: The test taker should notice the word "early" in the stem of the question and choose the option with the least amount of symptoms.

The nurse is teaching the client diagnosed with uterine prolapse. Which information should the nurse include in the discussion? 1. Increase fluids and daily exercise to prevent constipation. 2. Explain there is only one acceptable treatment for uterine prolapse. 3. Instruct the client to check the uterine prolapse visually every day. 4. Discuss limiting coughing and lifting heavy objects.

Answer: 4 1. Uterine prolapse is not caused by constipation; it is caused by a weakening of the pelvic muscles. It is a protrusion of the uterus through the vagina. It can pull on the vaginal wall, bladder, and rectum. 2. There are multiple treatment modalities for uterine prolapse. The selection of treatment is determined by the degree of the prolapse and the medical history of the client. 3. The protrusion from the vagina can be seen in some cases, but this is not so with all clients. 4. Symptoms can be aggravated by coughing, sneezing, lifting heavy objects, standing for prolonged periods, and climbing stairs. TEST-TAKING HINT: The test taker should eliminate options containing words such as "only," "never," "always," "all," or "most of the time." If the test taker does not have a clue, "uterine" is not in the same body system as constipation, so option "1" can be eliminated.

The gravida 7 para 6 client delivered a 9-pound 4-ounce infant 2 hours ago. Which intervention is a priority for the nurse to implement? 1. Assess the client's fundus every hour. 2. Assess the client's voiding pattern every shift. 3. Discuss birth control options with the client. 4. Discuss breastfeeding methods with the client.

Answer: 1 1. A gravida 7 para 6 client has had seven pregnancies and carried six of those pregnancies to 20 weeks or longer. This woman is at risk of having the fundus remain boggy and not diminishing in size, resulting in excessive bleeding. The nurse should assess the fundus hourly. 2. Voiding should be assessed related to the delivery time and position of the fundus, not just every 12 hours (a normal shift). 3. It is not a priority immediately postdelivery to discuss birth control options. 4. If the client wishes to breastfeed her infant at this time, with her history she can perform this without teaching. TEST-TAKING HINT: The test taker could eliminate option "2" because of the words "every shift." Timing words such as "hourly," "every day," "every 2 hours" can make an option correct or incorrect. The stem of the question said the delivery was 2 hours ago; this requires the nurse to determine which intervention should be performed immediately postdelivery.

The client has undergone a partial mastectomy for cancer of the left breast. Which discharge instruction should the nurse teach? 1. Don't lift more than 5 pounds with the left hand until released by the HCP. 2. The cancer has been totally removed and no follow-up therapy will be required. 3. The client should empty the closed wound suction evacuator drain about every 12 hours. 4. The client should arrange an appointment with a plastic surgeon for reconstruction.

Answer: 1 1. A partial mastectomy or excisional biopsy is a type of surgery on this side of the body. Pressure on the incision should be limited until the client is released by the HCP to perform normal daily activities. 2. This is providing the client with false hope. Cancer cells characteristically move easily in the lymph or bloodstream to other parts of the body. Microscopic disease cannot be determined by the naked eye. 3. A client after a mastectomy might be discharged with a closed wound suction evacuator (Hemovac) drain, but a partial mastectomy should not require one. 4. The breast has not been removed; reconstruction is not needed. TEST-TAKING HINT: If the test taker did not know this answer, option "1" is information provided to any client after surgery on the upper chest or arm.

The client scheduled to have a breast biopsy with sentinel node dissection states, "I don't understand. What does a sentinel node biopsy do?" Which scientific rationale should the nurse use to base the response? 1. A dye is injected into the tumor and traced to determine the spread of cells. 2. The surgeon removes the nodes that drain the diseased portion of the breast. 3. The nodes felt manually will be removed and sent to pathology. 4. A visual inspection of the lymph nodes will be made while the client is sleeping.

Answer: 1 1. A sentinel node biopsy is a procedure in which a radioactive dye is injected into the tumor and then traced by instrumentation and color to try to identify the exact lymph nodes the tumor could have shed into. The sentinel node is removed and tested for cancer cells. 2. This is the older procedure in which the surgeon removed the nodes thought to drain the tumor. There was no way of knowing if the surgeon was actually removing the affected nodes. 3. The purpose of the procedure is not to rely on guesswork in determining the extent of tumor involvement. 4. Microscopic disease cannot be seen by the naked eye. TEST-TAKING HINT: The test taker could eliminate options "3" and "4" if aware of the sentinel definition, which means "to watch over as a sentry." This might lead the test taker to determine what specific areas would have to be identified.

The nurse is caring for a client diagnosed with epididymitis secondary to a chlamydia infection. Which discharge instruction should the nurse discuss? 1. The sexual partner must be prescribed antibiotics. 2. Delay sexual intercourse for a minimum of 3 months. 3. Expect the urine to have white clumps for 1 to 2 months. 4. Drainage from the scrotum is fine as long as there is no fever.

Answer: 1 1. Chlamydia is a sexually transmitted infection usually silent in the male partner, but it can cause epididymitis. If both sexual partners are not treated, then the partner can reinfect the client. 2. Sexual intercourse can be resumed within a couple of weeks as long as both partners have been treated for the infection. 3. This indicates a urinary tract infection. A UTI is another cause of epididymitis. 4. The scrotum is a closed cavity. There should not be any drainage from the scrotum. If there is drainage, this could indicate a fistula and requires immediate notification of the HCP. TEST-TAKING HINT: The test taker should always look at the time frame provided in either the stem or the answer options as a clue to answering the question correctly. The test taker should look closely at any option that has a "month" time frame.

Which could be a complication of cryotherapy surgery for cancer of the prostate? 1. The urethra could become scarred and cause retention. 2. The client could have ejaculation difficulties and be impotent. 3. Bone marrow suppression could occur from the chemotherapy. 4. Chronic vomiting and diarrhea causing electrolyte imbalance.

Answer: 1 1. Cryotherapy involves placing freezing probes into the prostate to freeze the cancer cells. An indwelling catheter is placed into the urethra, and warm water is circulated through the catheter to try to prevent the urethra from freezing. If the urethra scars, then the lumen will constrict, causing retention of urine. 2. Ejaculation difficulties are caused by obstruction from tumor growth. 3. Bone marrow suppression is caused by radiation or chemotherapy. 4. Cryotherapy does not cause chronic vomiting or diarrhea. TEST-TAKING HINT: The test taker should dissect the word "cryotherapy." Cryo- means "to freeze," so cryotherapy indicates some form of cold therapy. Then the test taker should decide what cold therapy could cause to happen in the area being treated.

The nurse is planning the care of a client diagnosed with pelvic inflammatory disease secondary to an STI. Which collaborative diagnosis is appropriate for this client? 1. Risk for infertility. 2. Knowledge deficit. 3. Fluid volume deficit. 4. Noncompliance.

Answer: 1 1. Determining and diagnosing the risk for infertility problems requires collaboration between the nurse and the HCP. 2. The nurse is required to teach a client. This is an independent action. 3. Fluid volume deficit is not an appropriate nursing diagnosis for this client. 4. Noncompliance is an independent nursing problem. TEST-TAKING HINT: The question requires the test taker to determine which are autonomous functions of the nurse. The nurse does not have the capability to prescribe fertility medications or treatments.

The client has undergone a bilateral orchiectomy for cancer of the prostate. Which intervention should the nurse implement? 1. Support the scrotal sac with a towel and apply ice. 2. Administer testosterone replacement hormone orally. 3. Encourage the client to place sperm in a sperm bank. 4. Have the client talk to another man with ejaculation dysfunction.

Answer: 1 1. Elevating a surgical site and applying ice will reduce edema to the area. 2. The testes have been excised to remove the majority of male hormones. Replacing the hormones negates the purpose of the surgery. 3. Sperm banking is encouraged for younger men wanting children in the future. Prostate cancer is a cancer diagnosed in older men, and sperm banking is not normally recommended. 4. Bilateral orchiectomy will render the client impotent but not with ejaculation dysfunction. TEST-TAKING HINT: The test taker should try to match the procedure to the answer option. The procedure removes hormone-producing ability, so option "2" could be eliminated because it reverses the effects of the procedure.

An older woman is diagnosed with pelvic relaxation disorder secondary to age-related changes. Which medication should the nurse expect to administer? 1. Estrogen. 2. Dinoprostone. 3. Progesterone. 4. Oxytocin.

Answer: 1 1. Estrogen, a hormone, changes the pelvic floor muscles and lining of the uterus and may help improve a pelvic relaxation disorder. 2. Dinoprostone (Cervidil), a cervical ripening agent, is used to prepare the cervix for the delivery of a baby. It causes the cervix to shorten, soften, and dilate. 3. Progesterone, a hormone, is given for implantation of a fertilized ovum. 4. Oxytocin (Pitocin), an oxytocic agent, causes the uterus to contract. It is used during the labor and delivery process. TEST-TAKING HINT: Options "2," "3," and "4" are similar because they are all used during pregnancy or delivery. If the test taker doesn't have a clue as to the correct answer, the test taker should attempt to determine what answer is different from all the others.

The nurse is instructing a 2-week postpartum client with red, tender breasts after trying to breastfeed the infant. Which should the nurse teach the client? 1. Be sure the baby empties each breast when feeding. 2. Apply a warm, moist pack to the breasts for comfort. 3. Apply rubbing alcohol to the breast to treat the infection. 4. The baby must be given formula because the mother cannot breastfeed.

Answer: 1 1. Failure to have the baby to express all of the milk produced will result in less milk produced at future feedings and for the breasts to become sore and inflamed. 2. Ice packs are usually applied to decrease the discomfort. 3. Rubbing alcohol will dry out the breast tissues and the infant should not feed after alcohol has been applied to the breast because of absorption into the infant's system. 4. The mother should be taught correct breastfeeding techniques, not to avoid breastfeeding. TEST-TAKING HINT: The test taker could eliminate option "3" because the infant could ingest the alcohol during feeding.

The nurse is assessing a 34-year-old female client diagnosed with fibrocystic breasts. Which question should the nurse ask the client during the assessment? 1. "Are your breasts more tender during your period?" 2. "Have you ever developed an allergy to chocolate?" 3. "Can you tell me more about your feelings of having fibrocystic breast changes?" 4. "Have you considered having a prophylactic mastectomy?"

Answer: 1 1. Mastalgia (breast pain and tenderness) is common during the menstrual cycle. This question assists the nurse in determining if the mastalgia is from the fibrocystic disease. 2. Some women find that avoiding chocolate and stimulants (caffeine) reduces the discomfort of fibrocystic breasts, but it is not considered an allergy. 3. This is a therapeutic response, not an interviewing question to be asked during the assessment of the disease. 4. This is not precancerous; there is no reason for a mastectomy. TEST-TAKING HINT: The test taker could eliminate option "3" because a therapeutic response was not needed; the nurse needs to ask a question that assists in gaining information about the disease being assessed.

The nurse is preparing an educational presentation for women in the community. Which primary nursing intervention should the nurse discuss regarding the development of ovarian cancer? 1. Instruct the clients not to use talcum powder on the perineum. 2. Encourage clients to consume diets with high-fat content. 3. Teach the women to have a lower pelvic sonogram yearly. 4. Discuss the need to be aware of the family history of cancer.

Answer: 1 1. Research has shown the use of talcum powder perineally increases the risk for developing ovarian cancer, although there is no explanation known for this occurrence. Other risk factors include a high-fat diet, nulliparity, infertility, older age (70 to 80 years), mumps before menarche, and family history of ovarian cancer. 2. Nurses should never encourage a high-fat diet. 3. Only clients in a high-risk category should have routine sonograms. The time frame for the high-risk group of clients is 6 months. This is not a primary intervention; early detection is secondary intervention. 4. This alerts the client to participate in activities detecting cancer early, a secondary intervention. TEST-TAKING HINT: The test taker could eliminate options "3" and "4" because of the word "primary" in the stem. Option "2" could be eliminated because of the recommendation of a high-fat diet.

The client in the gynecology clinic asks the nurse, "What are the risk factors for developing cancer of the cervix?" Which statement is the nurse's best response? 1. "The earlier the age of sexual activity and the more partners, the greater the risk." 2. "Eating fast foods high in fat and taking birth control pills are risk factors." 3. "A Chlamydia trachomatis infection can cause cancer of the cervix." 4. "Having yearly Pap smears will protect you from developing cancer."

Answer: 1 1. Risk factors for cancer of the cervix include sexual activity before the age of 20 years; multiple sexual partners; early childbearing; exposure to the human papillomavirus; HIV infection; smoking; and nutritional deficits of folates, beta carotene, and vitamin C. 2. High-fat diets place clients at risk for some cancers but not for cervical cancer. The use of birth control pills may allow increased sexual freedom because of the protection from pregnancy, but it does not increase the risk for cancer of the cervix. 3. Infections with the human papillomavirus are a risk factor for cancer of the cervix. 4. Having a yearly Pap smear increases the chance of detecting cellular changes early, but it does not decrease the risk of developing cancer. TEST-TAKING HINT: The test taker could discard option "4" as a possible answer because it is a yearly test for the early detection of cervical cancer, not a risk factor.

Which is the American Cancer Society's recommendation for the early detection of cancer of the prostate? 1. A yearly PSA level and digital rectal exam (DRE) beginning at age 50. 2. A biannual rectal examination beginning at age 40. 3. A semiannual alkaline phosphatase level beginning at age 45. 4. A yearly urinalysis to determine the presence of prostatic fluid.

Answer: 1 1. The American Cancer Society recommends all men have a yearly PSA blood level test, followed by a DRE beginning at age 50. Men in the high-risk group, including all African American men, should begin at age 45. 2. A biannual (every 6 months) examination is not recommended. 3. Alkaline phosphatase levels are performed on men with known prostate cancer to determine bone involvement. This is not a screening test. 4. This test is done if the client has clinical manifestations of prostatitis. TEST-TAKING HINT: The nurse must be aware of recommended screening guidelines for a number of diseases. The test taker should carefully look at the time frame for the tests and the age of the client.

Which question is most important for the nurse to ask the client diagnosed with a cystocele and scheduled to have a pessary inserted? 1. "Do you know if you are allergic to latex?" 2. "When did you start having incontinence?" 3. "When was your last bowel movement?" 4. "Are you experiencing any pelvic pressure?"

Answer: 1 1. The client should be assessed for allergies to latex as a result of the composition of the pessary. 2. These clients frequently have incontinence, which is assessed before recommending a pessary. 3. A pessary is manually inserted to keep a prolapsed uterus in place. Asking about a bowel movement is not an appropriate question. 4. This is a symptom experienced by a client diagnosed with a cystocele. TEST-TAKING HINT: If the test taker has no idea what the answer is, then the test taker should apply Maslow's hierarchy of needs, which in this question is safety. Only option "1" addresses a safety issue. Allergies are a safety issue.

The client scheduled for chemotherapy is diagnosed with inflammatory breast cancer. Which intervention should the nurse implement based on the complete blood count (CBC) report? - RBCs: 4.8 - Hgb: 12.4 - Hct: 40 - WBCs: 7.8 - Platelets: 192 - ANC: 1500 1. Administer the chemotherapy as prescribed. 2. Notify the oncologist immediately. 3. Request urine and blood cultures. 4. Prepare to administer 1 unit of packed red blood cells.

Answer: 1 1. The nurse should administer the chemotherapy as ordered because all listed laboratory values are within normal limits (WNL). 2. The laboratory data are WNL; the oncologist can see the laboratory report on rounds. 3. The laboratory data are WNL; there is no need for cultures because infection is not indicated. 4. The laboratory data are WNL; no blood is needed. TEST-TAKING HINT: The test taker should be able to read and interpret reports so appropriate nursing decisions can be made.

The nurse is caring for a client newly diagnosed with stage IV ovarian cancer. What is the scientific rationale for detecting the tumors at this stage? 1. The client's ovaries lie deep within the pelvis and early symptoms are vague. 2. The client has regular gynecological examinations and this helps with detection. 3. The client had a history of dysmenorrhea and benign ovarian cysts. 4. The client had a family history of breast cancer and was being checked regularly.

Answer: 1 1. The ovaries are anatomically positioned deep within the pelvis, and because of this, clinical manifestations of cancer are vague and nonspecific. Clinical manifestations include increased abdominal girth, pelvic pressure, indigestion, bloating, flatulence, and pelvic and leg pain. Increasing abdomen size as a result of the accumulation of fluid is the most common clinical manifestation. Many women ignore the clinical manifestations because they are so nonspecific. 2. Regular gynecological examinations are recommended, but this is an advanced disease. 3. Dysmenorrhea is not a risk factor for developing ovarian cancer. Any enlarged ovary should be evaluated, especially if the client is postmenopausal, when the ovaries shrink in size. 4. A family history of breast cancer is a cause for the client to be assessed regularly for breast and ovarian cancer, but this is a late disease. TEST-TAKING HINT: "Stage IV" should help the test taker to eliminate options "2" and "4" because this client has advanced disease and it is hoped regular checkups find problems early.

Which statement best describes the responsibility of the public health nurse regarding STIs? 1. Notify the sexual partners of clients diagnosed with an STI. 2. Determine the course of treatment for clients diagnosed with an STI. 3. Explain the legal aspects of STI reporting to a client diagnosed with an STI. 4. Analyze the statistics regarding STI transmission and reporting the findings.

Answer: 1 1. The public health nurse is responsible for attempting to notify the sexual partners of a client diagnosed with an STI of potential infection and urging the partner to be tested for the disease and to receive treatment. Health departments offer confidential testing and treatment. 2. An HCP will determine the course of treatment for a client diagnosed with an STI. 3. The nurse can teach some information about reporting, but the nurse is not qualified to discuss all the legal aspects of reporting an STI. 4. The nurse is not responsible for analyzing statistics. TEST-TAKING HINT: Answer options "2," "3," and "4" ask the nurse to take on roles not within the nurse's expertise. The nurse must know the Nurse Practice Act of the state where the nurse practices. No state allows the nurse to give legal or medical advice.

The male client presents to the public health clinic reporting joint pain and malaise. On assessment, the nurse notes a rash on the trunk, palms of the hands, and soles of the feet. Which action should the nurse implement next? 1. Determine if the client has had a chancre sore within the last 2 months. 2. Ask the client how many sexual partners he has had in the past year. 3. Refer the client to a dermatologist for a diagnostic work-up. 4. Have the client provide a clean, voided midstream urine specimen.

Answer: 1 1. These are clinical manifestations of second-stage syphilis. The nurse should ask about the development of a chancre sore, one of the first signs of a syphilis infection. 2. This may be required of the public health nurse for notification of the partners, but it is not required to assess this problem. 3. This client does not need a dermatologist to determine an STI infection. The HCP can treat this infection. 4. A urine culture will not diagnose this disease. TEST-TAKING HINT: If the test taker is aware the clinical manifestations are those of an STI, options "3" and "4" can be eliminated.

The client diagnosed with cancer of the testes calls and tells the nurse he is having low back pain that does not go away with acetaminophen. Which action should the nurse implement? 1. Ask the client to come in to see the HCP for an examination. 2. Tell the client to use an NSAID instead. 3. Inform the client this means the cancer has metastasized. 4. Encourage the client to perform lower back strengthening exercises.

Answer: 1 1. This information could signal the onset of clinical manifestations of metastasis to the retroperitoneum. The HCP should see the client and discuss follow-up diagnostic tests. 2. This information should be investigated and not put off. 3. This may or may not have occurred. Only diagnostic tests can confirm metastasis. 4. Low back exercises will not help the client if this is metastatic cancer, and they could increase the pain. TEST-TAKING HINT: The nurse cannot diagnose the client as having metastasis, so option "3" can be eliminated. As a rule, any pain unrelieved with pain medication requires the client to see the HCP.

The client, who has had a mastectomy, tells the nurse, "My spouse will leave me now because I am not a whole woman anymore." Which response by the nurse is most therapeutic? 1. "You're afraid your spouse will not find you sexually appealing?" 2. "Your spouse should be grateful you will be able to live and be with him." 3. "Maybe your spouse would like to attend a support group for spouses." 4. "You don't know that is true. You need to give your spouse a chance."

Answer: 1 1. This is restating the client's feelings and is a therapeutic response. 2. This is not recognizing the client's concerns and putting the nurse's expectations on the spouse. 3. This is problem-solving and could be offered, but the therapeutic response is to restate the client's feelings and encourage a conversation. 4. The client may know this is true. The nurse is telling the client she has no reason for her feelings. Feelings are what they are and should be accepted as such. TEST-TAKING HINT: When the question asks for a therapeutic response, the test taker should choose an option encouraging the client to verbalize her feelings.

Which should the nurse teach the client regarding Breast Health Awareness (BHA) according to the American Cancer Society (ACS). Select all that apply. 1. Women at high risk should talk to the HCP about when to have a mammogram. 2. Beginning at age 45, the client should have a yearly mammogram. 3. The client should perform a breast self-examination (BSE) bimonthly. 4. The client should get a sonogram of the breasts semiannually. 5. The client should have an MRI of the breasts every 5 years.

Answer: 1, 2 1. Women at high risk for breast cancer should have the opportunity to have mammograms beginning earlier than recommended for routine screening. 2. According to 2015 guidelines, routine screening mammograms should begin at age 45 and be performed yearly until age 55. At age 55, mammograms should be performed biennially. 3. Beginning in their 20s, women should be told about the benefits and limitations of breast self-examination (BSE). Even those choosing not to do BSE should be aware of how their breasts normally look and feel and report any new breast changes to a health professional as soon as they are found. Finding a breast change does not necessarily mean there is cancer. BSE is performed monthly, not bimonthly. 4. Ultrasound or sonogram of the breast may be used to evaluate a breast change seen on a mammogram but are not performed semiannually on a routine basis. 5. MRIs are performed on women with special needs (breast implants, etc.) but are not performed on a routine basis. TEST-TAKING HINT: The test taker could eliminate option "3" based on the timing "bimonthly."

The nurse is assessing the client diagnosed with a rectocele. Which clinical manifestations should the nurse expect? Select all that apply. 1. Rectal pressure. 2. Flatus. 3. Fecal incontinence. 4. Constipation. 5. Urinary frequency.

Answer: 1, 2, 3, 4 1. A rectocele causes the rectum to be pouched upward, causing rectal pressure. 2. When the rectum pushes against the posterior wall of the vagina, the result is flatus. 3. Clients diagnosed with a rectocele experience fecal incontinence. 4. Clients diagnosed with a rectocele frequently are constipated. 5. A client diagnosed with a rectocele does not experience urinary frequency. TEST-TAKING HINT: The test taker should learn the terms. If the test taker does not know the answer, the test taker can eliminate urinary frequency because it is different from the other options. It refers to the urinary tract, whereas the others refer to bowel elimination.

The registered nurse (RN) and a UAP are caring for clients on a gynecology surgery floor. Which intervention can be delegated to the UAP? Select all that apply. 1. Empty the indwelling catheter on the 3-hour postoperative client. 2. Assist the client 2 days post-hysterectomy to the bathroom. 3. Monitor the peri-pad count on a client diagnosed with fibroid tumors. 4. Encourage the client refusing to get out of bed to walk in the hall. 5. Record the breakfast intake of the client preparing for discharge.

Answer: 1, 2, 4, 5 1. The UAP can empty the indwelling catheter and record the output. 2. This is an appropriate assignment. 3. Monitoring a peri-pad count is done to determine if the client is bleeding excessively; the nurse should do this as part of the assessment. 4. All personnel should encourage the client to ambulate. 5. This is an appropriate assignment. TEST-TAKING HINT: The RN cannot delegate assessment. The nurse must be aware of the tasks a UAP can perform.

The client diagnosed with cancer of the uterus is scheduled to have radiation brachytherapy. Which precautions should the nurse implement? Select all that apply. 1. Place the client in a private room. 2. Wear a dosimeter when entering the room. 3. Encourage visitors to come and stay with the client. 4. Plan to spend extended time with the client. 5. Notify the nuclear medicine technician.

Answer: 1, 2, 5 1. Brachytherapy is the direct implantation of radioactive seeds through the vagina into the uterus. The client should be in a private room at the end of the hall to prevent radiation exposure to the rest of the unit. 2. Nurses wear a dosimeter registering the amount of radiation they have been exposed to. When a certain level is reached, the nurse is no longer allowed to care for clients undergoing internal radiation therapy. 3. Visitors are limited while the radiation is in place. 4. In this case, spending extra time with a client is not done. The nurse only does what must be done and leaves the room. 5. The nuclear medicine technician will assist with the placement of the implants and will deliver the implants in a lead-lined container. The technician will also scan any items (linens and wastes) leaving the room for radiation contamination. TEST-TAKING HINT: This is an alternate-type question, which requires the test taker to select more than one correct answer. The test taker must select all the correct answers to receive credit for the question.

The client is being discharged after a left partial mastectomy. Which discharge instructions should the nurse include? Select all that apply. 1. Notify the HCP of a temperature of 100°F. 2. Carry large purses and bundles with the right hand. 3. Do not go to church or anywhere with crowds. 4. Try to keep the arm as still as possible until seen by the HCP. 5. Have a mammogram of the right and left breasts yearly.

Answer: 1, 2, 5 1. It is a common instruction for any client after surgery to notify the HCP if a fever develops. This could indicate a postoperative infection. 2. The client after a mastectomy is at risk for lymphedema in the affected arm because the lymph nodes are removed during the surgery. The client should protect the arm from injury and carry heavy objects with the opposite arm. 3. The client can attend church services and large gatherings. This client had surgery, not chemotherapy, which would make the client immunosuppressed. 4. The client should be taught arm-climbing exercises before leaving the hospital to facilitate maintaining range of motion. 5. The client has developed a malignancy in one breast and is at a higher risk for developing another tumor in the remaining breast area. TEST-TAKING HINT: The test taker must determine if the option of keeping the arm still is recommended. Most postoperative recommendations require the client to move as much as possible.

The nurse is caring for a client 1 day postoperative hysterectomy for cancer of the ovary. Which nursing interventions should the nurse implement? Select all that apply. 1. Assess for calf enlargement and tenderness. 2. Turn, cough, and deep breathe every 6 hours. 3. Assess pain on a 1-to-10 pain scale. 4. Apply sequential compression devices to legs. 5. Assess bowel sounds every 4 hours.

Answer: 1, 3, 4, 5 1. All clients after surgery are at risk for developing DVT, and an enlarged, tender calf is a clinical manifestation of DVT. 2. The client should be turned and encouraged to cough and deep breathe at least every 2 hours. 3. Clients after surgery should be assessed for pain on a pain scale and by observing for physiological markers indicating pain. 4. Sequential compression hose are used prophylactically to prevent DVT. 5. The client should be assessed for the return of bowel sounds. TEST-TAKING HINT: Option "2" has a time frame in it, and the test taker should ask if the time frame is correct for the intervention.

Which information should the nurse include in the discharge teaching for the client recovering from a laparoscopic hysterectomy? Select all that apply. 1. The client should report any bright red vaginal bleeding to the surgeon. 2. The client should start a vigorous exercise routine to restore her muscle tone. 3. The client should continue sitting in the bedside chair at least 6 hours daily. 4. The client should soak in a warm tub bath each night for 1 hour. 5. The client should drink six to eight glasses of water a day.

Answer: 1, 5 1. The client should report any bright red vaginal bleeding or bleeding that soaks more than one sanitary pad an hour. 2. Clients should rest and not start a vigorous exercise program until the surgeon gives permission. 3. Clients should avoid prolonged sitting to prevent blood clots. 4. Clients should avoid taking baths to help prevent infection of the incision site. 5. The client should drink 6 to 8 glasses of water a day to avoid constipation. TEST-TAKING HINT: When the test taker is selecting possible correct answers, the test taker should carefully consider the descriptive words. In option "2," the adjective "vigorous" should cause the test taker to eliminate it, and in option "3" the words "6 hours" should help eliminate it.

The nurse is caring for a client 8 hours postoperative unilateral orchiectomy for cancer of the testes. Which interventions should the nurse implement? Select all that apply. 1. Provide an athletic supporter before ambulating. 2. Encourage the client to delay the use of pain medications. 3. Place the client on a clear liquid diet for the first 48 hours. 4. Monitor the PT/INR levels and have vitamin K ready. 5. Use ice packs to the scrotum and around the incision.

Answer: 1, 5 1. The scrotum will require support during ambulation. An athletic supporter is designed to provide support in this area. 2. The client should be encouraged to take pain medications before the pain is at a high level to help the pain medication be more effective. 3. The client can be on a regular diet as soon as the client is not nauseated from the anesthesia. 4. This surgery does not increase bleeding times. 5. Ice packs will help to reduce swelling in the scrotum and alleviate pain. TEST-TAKING HINT: Option "2" can be eliminated because it states the opposite of the instructions given to clients about pain medication.

The female client presents to the gynecologist's office for the fifth time with an ovarian cyst and is scheduled for an exploratory laparoscopy. The client asks the nurse, "Why do I need to have another surgery? The other cysts have all been benign." Which statement is the nurse's best response? 1. "Because eventually, the cysts will become cancerous." 2. "All abnormal findings in the ovary should be checked out." 3. "The surgery will not be painful and you will have peace of mind." 4. "Are you afraid of having surgery? Would you like to talk about it?"

Answer: 2 1. All cysts do not become cancerous; 98% of ovarian cysts in clients younger than age 29 are benign, whereas in women older than age 50, about half are benign. 2. Any abnormal ovary that cannot be diagnosed with a transvaginal ultrasound should be examined laparoscopically. 3. Any time the client has surgery, she should be prepared to experience some pain. This is a false statement and could cause a breach in the nurse-client relationship. 4. This is a therapeutic statement and the client is asking for information. TEST-TAKING HINT: The test taker should read the stem of the question carefully. Option "1" has a form of absolute, "eventually will become," so it can be eliminated. Option "3" is a false statement. Option "4" is a therapeutic response and the stem asks the nurse to provide information.

The client is diagnosed with benign uterine fibroid tumors. Which question should the nurse ask to determine if the client is experiencing a complication? 1. "How many periods have you missed?" 2. "Do you get short of breath easily?" 3. "How many times have you been pregnant?" 4. "Where is the location of the pain you are having?"

Answer: 2 1. Benign fibroid tumors in the uterus cause the client to bleed longer with a heavier flow, not miss periods. 2. Many women delay surgery until anemia has occurred from the heavy menstrual flow. A symptom of anemia is shortness of breath. 3. The number of pregnancies does not matter at this time; the client has a different problem. 4. The pain is in the pelvic region to the low back, where the uterus lies. TEST-TAKING HINT: This is a high-level question requiring the test taker to make several judgments before arriving at the answer. First, the test taker must decide what happens when a client has fibroid tumors and then which symptoms the client will exhibit.

The nurse is admitting a client diagnosed with stage Ia cancer of the cervix to an outpatient surgery center for a conization. Which data would the client most likely report? 1. Diffuse watery discharge. 2. No symptoms. 3. Dyspareunia. 4. Intense itching.

Answer: 2 1. Diffuse, watery, foul-smelling discharge occurs at a much later stage. 2. At this stage the client is asymptomatic and cancer has been determined by a Pap smear. 3. Dyspareunia is painful sexual intercourse; the client is asymptomatic. 4. Intense itching occurs with vaginal yeast infections. TEST-TAKING HINT: The test taker could either choose option "2" because it is the least presenting symptom or discard it. Staging for all cancers starts with "0" or "1," indicating the least detectable cancer.

Which client has the highest risk for developing cancer of the testicles? 1. The client diagnosed with epididymitis. 2. The client born with cryptorchidism. 3. The client diagnosed with an enlarged prostate. 4. The client diagnosed with hypospadias.

Answer: 2 1. Epididymitis is inflammation of the epididymis, usually caused by bacteria descending from the prostate or bladder. This does not increase the risk of developing cancer of the testes. 2. Cryptorchidism is the medical term for an undescended testicle. The testicles may be in the abdomen or inguinal canal at birth. This condition places the client at higher risk for testicular cancer. 3. Prostate enlargement occurs as men age, whereas the most common age range for testicular cancer is age 15 to 35. 4. Hypospadias is a congenital abnormality in which the urethral meatus is on the underside of the penis. This does not increase the risk of cancer. TEST-TAKING HINT: This is a knowledge-based question, but answer options "3" and "4" could be eliminated based on anatomical position.

Which nursing task could be delegated to the unlicensed assistive personnel (UAP) for the client after a total vaginal hysterectomy? 1. Observe the color and amount of drainage on the client's perineal pad. 2. Maintain a current intake and output for the client each shift. 3. Provide the client with a plan of pharmacological pain management. 4. Prepare the client for her discharge scheduled for the next day.

Answer: 2 1. Observation is assessment, which cannot be delegated. 2. This nursing task can be delegated, but evaluation is the responsibility of the nurse. 3. Teaching cannot be delegated. 4. Teaching cannot be delegated. TEST-TAKING HINT: The RN cannot delegate assessment, teaching, evaluation, or an unstable client to a UAP.

The 67-year-old male client reports difficulty initiating a urinary stream, urinary frequency, and inability to completely empty the bladder. Which procedure would the nurse anticipate the HCP performing first? 1. Digital rectal examination (DRE). 2. Prostate-specific surface antigen (PSA). 3. Prostate ultrasound. 4. Biopsy of the prostate.

Answer: 2 1. The DRE is performed after the PSA to avoid false elevation of the PSA from the manipulation of the prostate during DRE. 2. The PSA is drawn before the DRE to prevent a false elevation due to the prostate being massaged during the DRE. 3. Prostate ultrasound is performed if a nodule is found or suspected and cannot be localized on DRE. It would not be anticipated as the first action by the HCP. 4. A biopsy would be done if all the other data are positive, but not first. TEST-TAKING HINT: The test taker could eliminate options "3" and "4" because they are more invasive than a laboratory test or a digital examination.

The school nurse is preparing a class on testicular cancer for male high school seniors. Which information regarding testicular self-examination should the nurse include? 1. Perform the examination in a cool room under a fan. 2. Any lump should be examined by an HCP as soon as possible. 3. Discuss having a second person confirm a negative result. 4. The procedure will cause mild discomfort if done correctly.

Answer: 2 1. The body temperature should be warm for the scrotum to relax. The best place to perform testicular self-examination is in a warm or hot shower. 2. The client may note a cordlike structure; this is the spermatic cord and is normal. Any lump or mass felt is abnormal and should be checked by an HCP as soon as possible. 3. The client can confirm his own negative result; a negative result is no masses felt. 4. The procedure is painless. If pain is elicited, then an HCP should examine the client. Cancer is usually painless; the presence of pain may indicate an infection. TEST-TAKING HINT: The test taker might choose option "2" as an answer because any abnormality should be examined by an HCP.

The client diagnosed with cancer of the prostate tells the nurse, "I caused this by being promiscuous when I was young and now I have to pay for my sins." Which statement is the nurse's most therapeutic response? 1. "Why would you think prostate cancer is caused by sex?" 2. "You feel guilty about some of your actions when you were young?" 3. "Well, there is nothing you can do about that behavior now." 4. "Have you told the HCP and been checked for an HIV infection?"

Answer: 2 1. The nurse should never ask "why." The client does not owe the nurse an explanation. 2. The question asks for a therapeutic response from the nurse. This response is restating and clarifying. 3. This is dismissive of the client's feelings and appears to agree with the client. There is no evidence cancer of the prostate is caused by promiscuous behavior. 4. This is a problem-solving statement and does not address the client's feelings. TEST-TAKING HINT: Because the question is asking for a therapeutic response, the test taker should address the client's feelings. However, the test taker should not use this test-taking hint as an absolute because each question should be answered on its own merit.

The 30-year-old male client diagnosed with germinal cell carcinoma of the testes asks the nurse, "What chance do I have? Should I end it all now?" Which response by the nurse indicates an understanding of the disease process? 1. "God does not want you to give up hope and end it all now." 2. "There is a good chance for survival with standard treatment options." 3. "There may be little hope, but ending it all is not the answer." 4. "You have a 50/50 chance of living for at least 5 years."

Answer: 2 1. The nurse should not impose any personal religious beliefs on the client. 2. Testicular cancers have very good prognoses, and even if the tumor returns, there is a good prognosis for extended survival. 3. There is a great deal of hope to offer these clients. 4. This is giving the client incorrect information. TEST-TAKING HINT: This is a question in which the test taker must know the information. Rarely can a nurse provide a specific percentage of survival.

The nurse is caring for a client diagnosed with uterine cancer receiving systemic therapy for 6 months. Which intervention should the nurse implement first? 1. Determine which antineoplastic medication the client has received. 2. Ask the client if she has had any problems with mouth ulcers at home. 3. Administer filgrastim to the client diagnosed with uterine cancer. 4. Encourage the client to discuss feelings about having cancer.

Answer: 2 1. This can be done to determine specific problems resulting from the specific side effects of the medication, but it is not the first action. The nurse can ask general assessment questions to determine how the client is tolerating the treatments. 2. The systemic side effects of chemotherapy are not always apparent, and the development of stomatitis can be extremely distressing for the client. The nurse should assess the client's tolerance to treatments. 3. The biologic response modifier filgrastim (Neupogen) is administered if the white blood cell count is low. The nurse assesses the WBC count and then obtains an order from the HCP. 4. This is an appropriate action, but not before assessing physical problems. TEST-TAKING HINT: When prioritizing nursing interventions, the test taker should apply the nursing process, and assessment is the first step.

The client diagnosed with cancer of the prostate has been placed on luteinizing hormone-releasing hormone (LHRH) agonist therapy. Which statement indicates the client understands the treatment? 1. "I will be able to function sexually as always." 2. "I may have hot flashes while taking this drug." 3. "This medication will cure prostate cancer." 4. "There are no side effects with this medication."

Answer: 2 1. This hormone will suppress the production of male hormones, and the client will not function sexually as usual. 2. The client may have hot flashes because these drugs increase hypothalamic activity, which stimulates the thermoregulatory centers of the body. 3. The medication decreases the growth rate of the cancer, but it does not cure the cancer. 4. There are side effects with all medications. These medications can cause gynecomastia, hot flashes, cardiovascular effects, and decreased sexual functioning. The LHRH agonists have fewer side effects than the estrogens. TEST-TAKING HINT: The test taker could eliminate options "3" and "4" because of the promise of no side effects or a cure.

The female client diagnosed with human papillomavirus (HPV) asks the nurse, "What other problems can HPV lead to?" Which statement is the most appropriate response by the nurse? 1. "HPV is transmitted during sexual intercourse." 2. "HPV infection can cause cancer of the cervix." 3. "It has been known to lead to ovarian problems." 4. "Regular Pap smears can help prevent problems."

Answer: 2 1. This is a true statement, but it does not answer the client's question. 2. An untreated HPV infection is a cause for developing cancer of the cervix. 3. HPV infection does not invade the abdominal cavity and therefore does not cause ovarian cancer. 4. The Pap test was developed to note early cell changes in the cervix. It indirectly monitors the effects of HPV, but it does not help prevent problems. TEST-TAKING HINT: The test taker should choose the answer for the question the client is asking. Option "1" discusses transmission and option "4" discusses prevention; therefore, these two options could be eliminated based on the stem of the question.

The female client's mother died from ovarian cancer, and her sister is now diagnosed with ovarian cancer. Which recommendations should the nurse make regarding early detection of ovarian cancer? 1. The client should consider having a prophylactic bilateral oophorectomy. 2. The client should have a transvaginal ultrasound and a CA-125 laboratory test every 6 months. 3. The client should have yearly magnetic resonance imaging (MRI) scans. 4. The client should have a biannual gynecological examination with flexible sigmoidoscopy.

Answer: 2 1. This is appropriate information if the client is in her mid- to late-30s and has completed her family, but this is not discussing early detection of ovarian cancer. 2. A transvaginal ultrasound is a sonogram in which the sonogram probe is inserted into the vagina and sound waves are directed toward the ovaries. The CA-125 tumor marker is elevated in several cancers. It is nonspecific, but coupled with the sonogram, can provide information about ovarian cancer for early diagnosis. 3. Yearly MRI scans will not provide the information the two tests will, and every 12 months is too long an interval. 4. A flexible sigmoidoscopy provides the HCP with a visual examination of the sigmoid colon, not the ovaries. TEST-TAKING HINT: The test taker could eliminate option "4" because of the anatomical site and option "3" because of the time factor. The test taker should ask, "If looking for early detection, at what interval should the client see the HCP?"

The client had a mastectomy for cancer of the breast and asked the nurse about a TRAM flap procedure. Which information should the nurse explain to the client? 1. The surgeon will insert a saline-filled sac under the skin to simulate a breast. 2. The surgeon will pull the client's own tissue under the skin to create a breast. 3. The surgeon will use tissue from inside the mouth to make a nipple. 4. The surgeon can make the breast any size the client wants the breast to be.

Answer: 2 1. This is done for reconstruction of a breast or augmentation of breast size, but it is not a TRAM flap procedure, which uses the client's own tissue. 2. The TRAM flap procedure is one in which the client's own tissue is used to form the new breast. Abdominal tissue, fat, skin, and blood vessels are moved under the skin to rebuild the breast. 3. The plastic surgeon can rebuild a nipple from pigmented skin donor sites or can tattoo the nipple in place. 4. This is true of saline implants but not of TRAM flaps. TEST-TAKING HINT: If the test taker is taking a standard pencil-and-paper test and is not familiar with this procedure, then skipping the question and returning to it at a later time is advisable. Another question might give a clue about the procedure. This is not possible on the NCLEX-RN® computerized examination.

The client diagnosed with uterine cancer is reporting lower back pain and unilateral leg edema. Which statement best explains the scientific rationale for these clinical manifestations? 1. This is the expected pain for this type of cancer. 2. This means the cancer has spread to other areas of the pelvis. 3. The pain is a result of the treatment of uterine cancer. 4. Radiation treatment always causes some type of pain in the region.

Answer: 2 1. This pain indicates metastasis in the retroperitoneal region. If caught early, a complete hysterectomy is usually the only therapy recommended. This type of pain indicates the cancer is advanced and the prognosis is poor. 2. This pain indicates the cancer is in the retroperitoneal region and the prognosis is poor. 3. Pain is not part of the treatment of cancer. Surgery may cause pain, but most treatments do not. 4. Radiation therapy does not always result in pain; it depends on the area irradiated. TEST-TAKING HINT: Option "4" has the absolute word "always" and should be eliminated as a correct answer. The stem is describing symptoms in regions other than the lower pelvis, so an educated choice is option "2."

The 50-year-old female client reports bloating and indigestion and tells the nurse she has gained 2 inches in her waist recently. Which question should the nurse ask the client? 1. "What do you eat before you feel bloated?" 2. "Have you had your ovaries removed?" 3. "Are your stools darker in color lately?" 4. "Is the indigestion worse when you lie down?"

Answer: 2 1. This statement would be appropriate if not for the abdominal girth change. This should alert the nurse to some internal reason for the change in girth. Ascites causes a change in abdominal girth. 2. Ovarian cancer has vague clinical manifestations of abdominal discomfort, but increasing abdominal girth is the most common symptom. If the client has had the ovaries removed, then the nurse could assess for another cause. 3. This could be an assessment for a peptic ulcer, but ulcers do not cause increasing abdominal girth. 4. This is a question to determine if the client has gastroesophageal reflux, but this does not cause an increased waist size. TEST-TAKING HINT: The test taker must notice all the symptoms the client is reporting. Flatulence and bloating could be associated with a number of problems, but these clinical manifestations, along with increased waist size, narrow the possibilities.

Which laboratory test should the nurse expect for the client to rule out the diagnosis of syphilis? 1. Vaginal cultures. 2. Rapid plasma reagin card test (RPR-CT). 3. Gram-stained specimen of the urethral meatus. 4. Immunological assay.

Answer: 2 1. Vaginal cultures are obtained to assess for gonorrhea and chlamydia. 2. The RPR-CT test and the Venereal Disease Research Laboratory (VDRL) test are diagnostic tests for syphilis. 3. Gram stains of the vagina or urethral meatus of a male may be done for gonorrhea but not for syphilis. 4. An immunological assay may be done for chlamydia but not for syphilis. TEST-TAKING HINT: The test taker must memorize the tests used to diagnose specific STIs and the clinical manifestations differentiating one STI from another.

The client is 4 months pregnant and finds a lump in her breast. The biopsy is positive for stage II cancer of the breast. Which treatment should the nurse anticipate the HCP recommending to the client? 1. A lumpectomy to be performed after the baby is born. 2. A modified radical mastectomy. 3. Radiation therapy to the chest wall only. 4. Chemotherapy only until the baby is born.

Answer: 2 1. Waiting until the baby is born allows the cancer to continue to develop and spread. This might be an option if the client were in the third trimester, but not at this early stage. 2. A modified radical mastectomy is recommended for this client because the client is not able to begin radiation or chemotherapy, which are part of the regimen for a lumpectomy or wedge resection. Many breast cancers developed during pregnancy are hormone sensitive and have the ideal grounds for growth. The tumor should be removed as soon as possible. 3. Radiation therapy cannot be delivered to a pregnant client because of possible harm to the fetus. 4. Chemotherapy is not given to the client while she is pregnant because of potential harm to the fetus. TEST-TAKING HINT: The test taker should eliminate options "3" and "4" because of potential harm to the fetus but also because each option has the word "only." There are very few "onlys" in health care.

The nurse is admitting a client diagnosed with trichomoniasis. Which assessment data support this diagnosis? 1. Odorless, white, curdlike vaginal discharge. 2. Strawberry spots on the vaginal surface and itching. 3. Scant white vaginal discharge and dyspareunia. 4. Purulent discharge from the endocervix and pelvic pain.

Answer: 2 1. An odorless, white, curdlike vaginal discharge is a symptom of Candida albicans, a vaginal yeast infection. 2. A strawberry spot on the vaginal wall or cervix, a fishy smelling vaginal discharge, and itching are clinical manifestations of trichomonas. 3. Scant white vaginal discharge and dyspareunia are clinical manifestations of atrophic vaginitis. 4. Purulent discharge from the endocervix and pelvic pain are clinical manifestations of cervicitis. TEST-TAKING HINT: When studying for a test covering similar diseases, the test taker should concentrate on the information that makes one different from another. Only one STI has a characteristic strawberry spot.

The nurse in the gynecology clinic is assessing a 14-year-old client reporting being sexually active. Which information should the nurse teach the client? Select all that apply. 1. Inform the client that the nurse must tell the parents of her being sexually active. 2. Teach the client about possible birth control options. 3. Instruct the client regarding sexually transmitted infections. 4. Demonstrate how a condom is applied correctly. 5. Tell the client about the importance of finishing all antibiotics. 6. Discuss the importance of attending parenting classes.

Answer: 2, 3, 4 1. The nurse does not have to inform the parents of the teenager's disclosure of information. 2. The nurse should discuss birth control and STIs with the client. She is at risk for pregnancy and STIs. 3. The nurse should discuss birth control and STIs with the client. She is at risk for pregnancy and STIs. 4. The male wears the most commonly used condoms, but both partners are responsible for contraception and prevention of STIs. This information will assist the client in knowing if the device is correctly applied and will have the best chance of preventing both pregnancy and STIs. 5. This would be information to provide if the client had an STI but is not needed at this time. 6. The client is not pregnant at this time. TEST-TAKING HINT: The test taker could eliminate options "5" and "6" because the client is not currently pregnant.

The client had a total abdominal hysterectomy and tumor debulking for endometrial cancer. Which discharge instruction should the nurse teach? Select all that apply. 1. The client should take hormone replacement therapy every day to prevent bone loss. 2. The client should practice pelvic rest until seen by the HCP. 3. The client can drive a car as soon as she is discharged from the hospital. 4. The client should expect some bleeding after this procedure. 5. The client should notify the HCP of fever or severe pain.

Answer: 2, 5 1. Clients diagnosed with cancer of the uterus have the ovaries removed to reduce hormone production. The client will not be taking HRT. 2. Pelvic rest means nothing is placed in the vagina. The client does not need a tampon at this time, but sexual intercourse should be avoided until the vaginal area has healed. 3. The sitting position a client assumes when driving a vehicle places stress on the lower abdomen. The client should wait until the HCP releases her to drive. 4. The client should not have any vaginal bleeding. 5. Clients should be instructed about signs of complications that should be reported including fever, severe pain, and urinary burning or frequency. TEST-TAKING HINT: The test taker should apply basic postoperative concepts when answering questions and realize bleeding is not expected postoperatively and safety should always be addressed.

Which steps should the nurse provide clients choosing to perform breast self-examination (BSE) according to the American Cancer Society (ACS) guidelines? Rank in order of performance. 1. Lie flat on the bed with a rolled towel placed under the scapula; perform palpation of each breast. 2. Pinch each nipple to see if fluid can be expressed. 3. With the breasts exposed, stand in front of a mirror and examine the breasts from the front and each side. 4. In the shower, soap the breasts, and perform palpation in a systematic manner on each breast. 5. Find a private place where the self-examination can be performed.

Answer: 5, 3, 4, 1, 2 5. BSE should be performed in a private area with good lighting and a comfortable temperature so the woman is warm and can be consistent in the steps. 3. The first part of BSE is performed in front of a mirror. The breasts are examined from the front and each side, then with arms down by the side, then raised, next with the arms on the hips and bending over. The woman is looking for dimpling and irregularity in size or shape. 4. The woman then gets into a warm shower and palpates each breast when the breasts are soapy and slippery. 1. The last position the woman assumes is lying flat on the bed with a towel behind each scapula and palpation is performed. 2. After each breast is palpated, the last thing done is to pinch the nipples to see if there is a discharge. TEST-TAKING HINT: The test taker must remember basic guidelines for procedures and which steps are involved.


Set pelajaran terkait

Ch. 4 Prenatal Development and Birth

View Set

Chapter 32: Assessment of Hematologic Function and Treatment Modalities

View Set